PA ER practice

Ace your homework & exams now with Quizwiz!

A 44-year-old female presents to the emergency department with a right sided headache. She states the headache is located on the right temple region, is non-radiating, and does not cause photophobia. She is otherwise healthy and has no reported medical problems and has only taken acetaminophen for the pain, which has minimal relief. On physical exam she is alert, awake, and oriented. Her vitals are T 98.8, P 78, R 18, and BP 128/76. Her head is normocephalic, atraumatic, and pupils are equal and reactive. She has tenderness to the right temporal area of the temporal artery, and there is no noted swelling, redness, or abnormalities noted. There is no tenderness to the cervical muscles. She exhibits a non-focal neurological exam. A biopsy of the temporal artery reveals inflammatory cells of the artery wall. Based on the history and biopsy results, what is the best initial treatment for this patient? A Cyclosporine B Azathioprine C Methotrexate D Ibuprofen E Prednisone

The Correct Answer is: E This patient is exhibiting a case of temporal arteritis. She has the signs and symptoms that are classic in nature and do not fit into the realm of the other headaches, all of which usually have different characteristics on history and physical exam. In this instance, the use of oral steroids will be the best choice in management of the condition. Anti-inflammatories (C, D) and immunosuppressant agents (A, B) will not be as effective as oral prednisone (E), and the long-term outcomes are improved with steroids.

An adult male, not previously vaccinated for rabies, presents to the emergency department after being bitten by an aggressive stray dog. The dog was captured, and declared "probably rabid" by a local veterinarian. Which of the following treatment options should you select for this patient? A administer human rabies immune globulin only B administer equine rabies antiserum only C administer human rabies immune globulin and equine rabies antiserum D administer human rabies immune globulin and equine rabies antiserum and human diploid cell rabies vaccine E administer human rabies immune globulin and human diploid cell rabies vaccine

The Correct Answer is: E Transmission of rabies to this patient must be seriously considered, and postexposure immunization should begin immediately by the administration of human rabies immune globulin (HRIG; 20 IU/kg). About half the HRIG should be infiltrated around the bite wound, and the remainder injected intramuscularly. Human diploid cell rabies vaccine (HDCV) should also be given (1 mL IM in the deltoid), and again on days 3, 7, 14, and 28. HDCV should be delivered in a different syringe and administered at a different site than HRIG.

A 77-year-old male is admitted to the ICU with community acquired pneumonia and sepsis. There is a concern for the possible development of stress gastritis. Which of the following is an important risk factor in the development of stress gastritis? A Platelets < 150,000 per microliter B INR < 1.5 C Patient remaining NPO for > 24 hours D Hematocrit < 35% E Respiratory failure requiring mechanical ventilation > 48 hours

The Correct Answer is: E Two of the most important risk factors in the development of stress gastritis are coagulopathy (platelets < 50,000/mcl or INR > 1.5) and respiratory failure, requiring mechanical ventilation for > 48 hours. Hematocrit and amount of time that a patient receives nothing by mouth are not important risk factors.

A 50-year-old woman with a history of hypertension complains of chest tightness and dyspnea while walking up one flight of stairs. She recently experienced an episode of near-syncope. She denies a history or rheumatic fever. On auscultation, a crescendo-decrescendo systolic ejection murmur is heard at the upper right sternal border radiating to the carotids bilaterally. Given the patient's physical exam findings, which of the following is the most likely diagnosis? A Severe aortic stenosis secondary to congenital bicuspid aortic valve B Aortic regurgitation/insufficiency C Mitral stenosis D Aortic regurgitation/insufficiency E Tricuspid regurgitation/insufficiency

A Choice A is correct, as the murmur of aortic stenosis is usually described as a crescendo-decrescendo or systolic ejection murmur heard best at the right upper sternal border. In addition, the murmur of aortic stenosis is frequently transmitted to the carotid arteries. In a patient of this age, with symptoms suggestive of severe aortic stenosis and with these physical exam findings, a congenital bicuspid aortic valve is the best choice, especially as the patient denies a history or rheumatic fever, which is also a cause of developing aortic valve stenosis in individuals under the age of 65. Patients with a congenital bicuspid aortic valve typically develop symptoms once the valve leaflets have become calcified and thickened, secondary to the undue stress over many years on a structurally abnormal aortic valve. Choice B is incorrect, as the murmur of aortic regurgitation is usually described as a high-frequency decrescendo early diastolic murmur heard best at the left upper sternal border or at the right upper sternal border. Choice C is incorrect, as the murmur of mitral stenosis is described as a low-frequency rumbling diastolic murmur that is decrescendo in early diastole, but may become crescendo up to the first heart sound with moderately severe mitral stenosis and sinus rhythm. Choice E is incorrect, as the murmur of tricuspid regurgitation is described as a holosystolic descrescendo murmur.

A 21-year-old male presents to the emergency department due to vomiting blood. He is a college student and was initiated into a fraternity earlier in the week, and admits to drinking enough alcohol to make him vomit that night. There was no blood in the vomit at that time. The hematemesis started today, about 20 minutes after he finished a workout at the gym. He notes that today was a particularly tough workout, as he increased the amount of weight he was lifting significantly compared to previous workouts. His vital signs are as follows: heart rate 86 bpm, respirations 14, BP 126/82, temp 98.8°F. What is his most likely diagnosis? A Mallory-Weiss syndrome B Peptic ulcer disease C Arteriovenous malformation D Esophageal varices E Erosive gastritis

A The correct answer is Mallory-Weiss syndrome. This is characterized by a nonpenetrating mucosal tear at the gastroesophageal junction, which may arise from a sudden increase in transabdominal pressure, such as lifting, retching, or vomiting. Peptic ulcer disease usually presents with a history of dyspepsia. Arteriovenous malformations usually present with melena or hematochezia. Esophageal varices usually develop secondary to portal hypertension, and are most often found in patients with cirrhosis. Erosive gastritis is often due to NSAIDs, alcohol, or severe medical or surgical illness, and is more often associated with chronic blood loss.

A 46-year-old female presents to the emergency department with a complaint of hematemesis, nausea, and epigastric pain. The hematemesis started earlier this morning, while the epigastric pain started three days prior and was a dull ache. She has vomited three times. She denies fever, diarrhea, or weight loss. She has not had blood in her stool. Her current medications include ibuprofen. She has been taking this medication daily for the past three months for right knee pain. What is the test that is the most sensitive for diagnosing this disease process? A Upper endoscopy B Hemoccult test C Esophageal manometry D Barium esophagography E Urea breath test

A The correct answer is upper endoscopy. The most likely diagnosis is gastritis, and upper endoscopy is the most sensitive method of diagnosis. Hemoccult testing is used to detect the presence of blood in the patient's stool. Esophageal monometry is used most often in establishing an etiology of dysphagia. Barium esophagography is also used frequently in the evaluation of patients with dysphagia. Neither esophageal manometry or barium esophagography would help you determine the source of this GI bleed. Urea breath test is used in detecting H. pylori infection, but would not be the best choice in diagnosing gastritis.

A teenage girl presents to the emergency department with her parents. She has had symptoms of a urinary tract infection for the last two days, but did not tell her parents until today. She is not sexually active. Today, the girl also has diffuse abdominal pain with vomiting, general malaise, and difficulty breathing. She has no significant past medical history. Her physical exam reveals sinus tachycardia and deep fast respirations with no localization of abdominal pain or rebound tenderness. Initial lab test results reveal a plasma glucose = 378 mg/dL and serum bicarbonate = 14 mEq/L. What is the most likely diagnosis? A Hyperosmolar nonketotic hyperglycemia B Diabetic ketoacidosis C Chronic corticosteroid use D Gestational diabetes E Schmidt syndrome

B The correct choice is B, diabetic ketoacidosis (DKA). Classic signs and symptoms of this disorder include polyuria, polydipsia, marked fatigue, nausea, vomiting, signs of dehydration, fruity breath odor, postural hypotension, Kussmaul respirations, and possibly mental stupor or coma. Patients with type 1 diabetes mellitus may present for the first time in DKA. DKA is commonly precipitated by a recent infection. Choice A, hyperosmolar nonketotic hyperglycemia, is seen in patients with type 2 diabetes and presents with extremely high plasma glucose levels without acidosis and ketosis. Choice C, chronic corticosteroid use, can cause hyperglycemia and possible glucose intolerance or diabetes mellitus. This patient has no history of oral corticosteroid use. Choice D, gestational diabetes, occurs in women who are pregnant. Choice E, Schmidt syndrome, is an autoimmune polyglandular syndrome which includes diabetes mellitus in greater than 50% of patients.

A 56-year-old male, with history of hyperlipidemia and non-insulin-dependent diabetes mellitus (NIDDM) presents to the emergency department with a history of increasing peripheral edema over the past week. On examination he is noted to have periorbital, scrotal, and +2 pretibial edema. His lungs are CTAB. He denies any chest pain or shortness of breath. Urine dipstick reveals 4+ protein. Urine microscopic reveals Maltese crosses consistent with lipiduria. Labs include a decreased serum albumin of 2 g/dl, decreased total protein of 5.5 g/dl, and normal glomerular filtration rate (GFR). What is the most likely diagnosis? A pyelonephritis B congestive heart failure (CHF) C nephrotic syndrome D prostatitis E Deep venous thrombosis (DVT)

C The correct answer is (C). This patient has typical symptoms of nephrotic syndrome, which includes significant proteinuria, hypoalbuminemia, and typical presentation of edema. He also has a history of hyperlipidemia and laboratory findings of lipiduria, which is also common in nephrotic syndrome. Furthermore, his history of diabetes mellitus is also a potential cause of nephrotic syndrome. Pyelonephritis and prostatitis would present with urine WBCs and is not consistent with the laboratory findings or edema. CHF would more likely present with dyspnea, rales on exam, and peripheral edema but would unlikely involve the periorbital area. DVT would likely present with unilateral swelling of the LE, and discomfort and is not consistent with the laboratory findings above.

You are evaluating a 67-year-old male with known cirrhosis of the liver secondary to alcoholic liver disease, although he has been sober for the past year. He is brought in to the emergency department by his daughter, who notes that for the past few days he has seemed to be more confused. On examination you note the patient to be mildly confused but alert to person and place. He has noticeable asterixis. He is not currently taking any medications and his blood alcohol level is undetectable. What is the treatment of choice in this case based on your physical examination findings? A amoxicillin B prednisone C lactulose D folic acid E thiamine

C The correct answer is (C). This patient most likely has hepatic encephalopathy due to end-stage liver disease. Asterixis indicates an increase in serum ammonia. The treatment of choice is lactulose. Both folic acid and thiamine are used in the treatment of alcoholic liver disease, but do not treat elevated ammonia levels. Antibiotics may be used secondarily in patients nonresponsive to lactulose, but amoxicillin is not preferred. Prednisone is not a treatment for hepatic encephalopathy.

A 76-year-old man with a history of three myocardial infarctions, CABG x 4, COPD, HTN, and hyperlipidemia presents to the emergency department with complaints of increasing dyspnea with exertion. He now has to sit in his recliner to sleep. He complains of lower extremity edema, now above the knees. On physical examination, he exhibits jugular venous distention to the angle of the jaw at 90 degrees of truncal elevation, diffuse bilateral rales on auscultation of the lung, and a blowing systolic murmur that is heard best at the left lower sternal border. Which abdominal physical exam finding would be most likely given this patient's history? A Periumbilical ecchymoses B Murphy's sign C Tenderness at McBurney's point D Hepatomegaly E Absent bowel sounds

D Choice D is correct, as the patient is demonstrating signs and symptoms of congestive heart failure, which can lead to hepatic congestion and hepatomegaly. Choice A, periumbilical ecchymoses, or Cullen's sign, are representative of hemoperitoneum. Choice B, Murphy's sign, is indicative of cholecystitis. Choice C, tenderness at McBurney's point, is frequently seen with appendicitis. Choice E, absent bowel sounds, is seen in patients with ileus.

A 14-year-old boy presents to the emergency department with his parents. He has a history of type 1 diabetes, and has had bronchitis for the last few days. He is now presenting with difficulty breathing, worsening fatigue, polydipsia, and polyuria. His last fingerstick glucose at home this morning was 350 mg/dL. Which of the following patterns of breathing are characteristic of this complication of diabetes? A Cheyne stokes respiration B Bradypnea C Biot breathing D Kussmaul breathing E Painful respiration

D The correct choice is D, Kussmaul breathing, which is deep regular breathing or hyperpnea. It can be seen as a compensatory action of metabolic acidosis and hypoxia. Choice A, Cheyne-Stokes respiration, is a waxing and waning pattern of rate and volume that includes periods of apnea. This can be seen in patients at high altitudes, and with severe left sided heart failure or neurologic disease. Choice B, bradypnea, is noted with a slower than usual respiratory rate and can be seen with use of CNS depressant drugs, uremia, or structural intracranial lesion. Choice C, Biot breathing, is an uncommon variant of Cheyne-Stokes respiration, with periods of apnea alternating with a series of equal breaths that end abruptly. It can be seen in patients with meningitis. Choice E, painful respiration, is relatively normal in pattern, but interrupted by pain during breathing from such disorders as pleurisy, fractured ribs, or subphrenic inflammation.

A 21-year-old man presents to the emergency department complaining of a swollen left ear (see image) after he experienced blunt trauma in a collegiate wrestling match. Which of the following is the most likely diagnosis? A Avulsion B Cellulitis C Contusion D Hematoma E Cauliflower ear

Explanation The Correct Answer is: D The history of blunt trauma resulting in acute swelling, ecchymosis, and loss of cartilaginous landmarks is consistent with auricular hematoma (D). An auricular contusion (C) would result in maintenance of the landmarks. The patient currently lacks cauliflower ear (E), but is at risk of developing this deformity as a result of this hematoma.

Which type(s) of Salter-Harris fractures can generally be treated with closed reduction and cast immobilization? A Type I B Types I and II C Types I, II, and III D Types I, II, III, and IV E Types I, II, III, IV, and V

The Correct Answer is: C Minimally displaced Salter-Harris types I, II, and III fractures generally can be treated with immobilization only. Types IV and V involve the cartilage of both the articular surface and the growth plate. To ensure proper alignment and a congruous joint surfaces open reduction and internal fixation is usually necessary.

Two days following an uneventful 4-vessel CABG, a 57-year-old man develops a sudden onset of lightheadedness and palpitations. His vital signs are stable, and physical examination demonstrates no abnormalities. Given the results of his EKG, as shown (Figure 2), which of the following is the most appropriate next step in management? A Direct-current cardioversion B Nitroglycerin patch C Digoxin 0.125 mg PO daily D Neurology consult E Meclizine 25 mg PO Q6H

The Correct Answer is: A Among the choices offered here, choice A is the most appropriate next step in management of a patient with new onset atrial flutter, as determined by EKG; it most effectively converts most patients to normal sinus rhythm. Choice B is inappropriate, as the patient is not demonstrating angina pectoris, and the EKG does not demonstrate evidence of ischemia or infarction. Choice C is inappropriate, as it is the least effective agent for slowing the ventricular response when compared to beta blockade or calcium channel blockers, all of which act by blocking the AV node (digixon may occasionally convert atrial flutter to atrial fibrillation). Choice D is inappropriate, as the patient's symptoms of lightheadedness do not stem from neurologic changes. Choice E is inappropriate, as the patient's symptoms do not stem from vertigo.

A 36-year-old male is brought to the emergency department in handcuffs, after he sustained a laceration to his left leg while trying to outrun the police. Apparently, he was driving a stolen car when he was pulled over for speeding. Upon talking with him, you learn that he has been in jail twice before for robbery. During the history and physical exam he appears callous and impulsive. What is the most appropriate classification for this personality disorder? A Antisocial B Avoidant C Narcissistic D Paranoid E Schizoid

The Correct Answer is: A Antisocial personality disorder consists of clinical findings that include selfishness, callousness, promiscuousness, impulsive behavior, and an inability to learn from experience and legal problems. Avoidant personality disorder presents clinically as someone who fears rejection, overreacts failure, has poor social endeavors, and low self-esteem. Narcissistic personality disorder presents with the clinical findings of grandiosity, a preoccupation with power, lacking interest in others, and excessive demands for attention. Clinical findings of someone who has paranoid personality disorder would include defensiveness, being overly sensitive, secretive, suspicious, and hyper-alert, and having a limited emotional response. Clinical findings of schizoid personality disorder include being shy, introverted, withdrawn, and avoiding close relationships.

A patient is brought to the Emergency Department by ambulance. He is a 27-year-old male who is well known to the paramedic team as a heroin addict. He is arousable and does not remain alert when aroused. Which of the following physical signs would help to confirm the diagnosis of opioid intoxication? A Bradycardia B Diaphoresis C Mydriasis D Rhinorrhea E Tachypnea

The Correct Answer is: A Bradycardia (A), lowered respiratory rate, miosis and somnolence are the main effects of opiates. As with most medications, withdrawal effects are the opposite of the effects of overdose. Tachycardia, tachypnea, rhinorrhea and diaphoresis (B, D, & E) can all occur in opiate withdrawal. Mydriasis (C) is common with other substances of abuse, such as cocaine and LSD.

A 48-year-old man is brought to the emergency department by his sister after suffering from loss of consciousness, followed by muscle rigidity and rhythmic contractions, and then a return to a normal state. When asked about medication use, the patient states he is currently being treated with a drug for depression but cannot remember the name. He claims that he has never had a seizure or seizure-like activity prior to this event. Approximately 6 hours after the first episode, the patient suffers a second one while still in the ED. Which of the following medications is the patient most likely taking? A bupropion B duloxetine C fluoxetine D nortriptyline E phenelzine

The Correct Answer is: A Bupropion has been shown in some patients to cause seizures in a dose-dependent fashion, particularly in those with a history of head trauma or electrolyte abnormalities. Tricyclic antidepressants (eg, nortriptyline), selective serotonin reuptake inhibitors (eg, fluoxetine), serotonin-norepinephrine reuptake inhibitors (eg, duloxetine), and monoamine oxidase inhibitors (eg, phenelzine) have not been associated with seizures.

A 24-year-old HIV-positive man comes to the emergency department complaining of severe left-sided chest discomfort, which radiates through to the left trapezius region. On coming into the room, you note that he is sitting up and hunched forward. On physical examination, the patient's blood pressure is 135/78, with a pulse of 85 bpm, and a pericardial friction rub is noted. Laboratory findings demonstrate elevated serum creatine kinase levels and normal serial troponin levels. His EKG demonstrates global ST segment elevation. His CXR demonstrates no acute process. Which of the following is the most likely diagnosis in this patient? A Acute pericarditis B Acute myocardial infarction C Acute bacterial endocarditis D Acute ascending aortic dissection E Acute costochondritis

The Correct Answer is: A Choice A is the most likely finding, as this patient is exhibiting signs, symptoms, and EKG findings pathognomonic for acute pericarditis, which is likely infectious in the setting of a patient with HIV. A pericardial friction rub is heard best with the patient in a seated position, during expiration, and is frequently found in patients with pericarditis. Choice B, an acute myocardial infarction, is less likely in a patient of this age, especially with normal serial troponins. Acute pericarditis can sometimes present with elevated serum creatine kinase levels when the epicardium is also involved. Choice C, acute bacterial endocarditis, is less likely in a patient with these EKG changes. Choice D, aortic dissection, would present with chest pain; however, the patient would be markedly hypotensive, less stable on presentation, and a CXR would demonstrate widening of the superior mediastinum.

Patients in which of the following age groups are least likely to experience a dislocation or sprain when a significant stress is placed on their joints? A 5-10 years old B 15-20 years old C 35-40 years old D 55-60 years old E 75-80 years old

The Correct Answer is: A Dislocations and ligamentous injuries are uncommon in prepubertal children as the ligaments and joints are quite strong as compared to the adjoining growth plates. Excessive force applied to a child's joint is more likely to cause a fracture through the growth plate than a dislocation or sprain.

A 27-year-old female presents to the emergency department with severe RUQ pain. Ultrasonography reveals gallstones. In preparation for a potential cholecystectomy, a CBC is obtained that reveals a normocytic anemia with an increased mean corpuscular hemoglobin concentration (MCHC). She is slightly jaundiced and you are able to palpate her spleen on examination. What is her underlying diagnosis? A Hereditary spherocytosis B Iron deficiency anemia C Sickle cell anemia D Thalassemia E Von Willebrand's disease

The Correct Answer is: A Hereditary Spherocytosis (HS) is characterized by jaundice, an enlarged spleen, and often gallstones; gallstones are more frequently seen in young people, triggering the investigation into HS. An increased MCHC is a characteristic feature of HS and is almost the only condition in which this finding is seen. Iron deficiency anemia does not have an increased MCHC. Sickle Cell has characteristic findings different than the presenting findings. Thalassemia is noted for a microcytosis and Von Willebrand's is a coagulation disorder.

A 36-year-old auto mechanic presents to the emergency department after hurting his back on the job. While lifting an object, he experienced sudden pain in his lower back with radiation to the right buttock. He was initially treated for muscle strain with a nonsteroidal anti-inflammatory drug (NSAID) after x-rays of his lumbosacral spine demonstrated no pathology. He continued to complain of this low back pain now radiating posteriorly down his left leg to the mid-thigh. Physical examination is unremarkable. The most likely diagnosis is A lumbosacral strain B left S1 radiculopathy C cauda equina syndrome D L5-S1 disc herniation E lateral femoral cutaneous neuropathy

The Correct Answer is: A Low back pain is one of the more common presenting neurologic complaints to a primary care provider. Most acute pain syndromes are benign, self-limiting conditions, with pain arising from myofascial sources. Patients with back pain and normal neurologic examinations are unlikely to have any serious underlying pathology and further diagnostic testing is usually unrevealing.

Which of the following is the drug of choice for acute hypertensive encephalopathy? A Labetolol B Clonidine C Furosemide D Nifedipine E Nitroglycerin

The Correct Answer is: A Neurologic emergencies associated with elevated blood pressure must be throurougly evaluated to determine the diagnosis and appropriate treatment plan. In the case of hypertensive encephalopathy, immediate attention must be focused on blood pressure reduction. Multiple agents are known to decrease blood pressure, but selection must also focus on how rapidly each agent works, how titratable the agent is, and any potential sequelae from using a particular agent. Sodium nitroprusside was classically the agent of choice for rapid blood pressure management, but it has fallen out of favor due to its monitoring requirements and rate of toxicity. Labetolol, a beta-blocker, is recommended for acute management, except in the case of cocaine intoxication. It has a rapid onset of action, can be titrated, and is given IV. Use of labetolol should be avoided in patients with known asthma, COPD, congestive heart failure, bradycardia, and second or third degree heart block. Additional agents appropriate for hypertensive encephalopathy include enalaprilat, esmolol, fenoldopam, hydralazine, and nicardipine, with each being considered carefully (based on patient condition and other factors). Agents with known CNS adverse effects, such as clonidine, should be avoided. Use of clonidine must also be monitored due to potential rebound hypertension. Nifedipine use is discouraged in hypertensive emergencies. Nitroglycerin should not be used for hypertensive encephalopathy because it increases intracranial pressure

A 32 year-old male presents to the emergency department in Acute Renal Insufficiency (AKI). Which of the following conditions would be most likely observed in intrinsic AKI? A Septic shock B Congestive heart failure C Benign prostatic hypertrophy D NSAID overdose E Chronic liver failure

The Correct Answer is: A The most common causes of intrinsic AKI are sepsis, ischemia, and nephrotoxins, both endogenous and exogenous. Prerenal acute kidney injury can be caused from hypovolemia, decreased cardiac output, decreased circulation of blood volume (CHF, liver failure), and impaired renal autoregulation (NSAIDs, ACE-I/ARB, cyclosporine)---(E), (D), and (B). Postrenal causes include bladder outlet obstruction including bladder stones and BPH (C).

A 32-year-old female is brought into the emergency department by her partner. His report indicates that she had been in her usual state of good health until a couple of days ago. At that time she started to complain of feeling fatigued. She now appears jaundiced and lethargic, and is complaining of chest pain. On exam, her spleen is palpable. Hemoglobin is 6 g/dl and she is Coombs positive. What is the most likely diagnosis? A Autoimmune hemolytic anemia B Glucose-6-phosphate dehydrogenase deficiency C Hereditary spherocytosis D Pyruvate kinase deficiency E Thalassemia

The Correct Answer is: A The onset of autoimmune hemolytic anemia (AHA) is often abrupt and dramatic. Anemia can develop in days, along with jaundice and splenic enlargement. When this triad is present, the suspicion for AHA must be high. The diagnostic test for AHA is the Coombs test. If positive, it confirms the presence of the antibody on the red cells. All other diseases listed are Coombs negative hemolytic anemias.

A 78-year-old woman with a medical history of diabetes and hypertension presents to the emergency department complaining of left hand weakness and slurred speech. Which of the following tests is most likely to determine the source of an arterial thrombus? A carotid ultrasound B CT of the brain C erythrocyte sedimentation rate D magnetic resonance angiography (MRA) of the vertebral arteries

The Correct Answer is: A The patient's symptoms are consistent with pathology arising from the anterior cerebral circulation including the carotid arteries. A CT should be ordered to rule out acute hemorrhage and an erythrocyte sedimentation rate may be useful if giant cell arteritis were suspected. An MRA of the vertebral arteries would likely show deficits but is not likely to demonstrate the etiologic location of this stroke.

A 23-year-old patient who has recently been on a ski trip presents with pain to the right hand after sustaining a fall. It is difficult to move, and there is pain on flexion of the digit. Based on this history what ligament would the patient most likely have injured? A 1st MCP joint ulnar collateral ligament B 2nd MCP joint ulnar collateral ligament C 3rd MCP joint ulnar collateral ligament D 4th MCP joint ulnar collateral ligament E 5th MCP joint ulnar collateral ligament

The Correct Answer is: A The ulnar collateral ligament at the base of the thumb, or 1st MCP joint, is often injured in forced abduction, such as a fall while skiing or during other sporting activities. An injury to this ligament has traditionally been called Gamekeeper's Thumb, but the origin of this term referred to a more chronic injury sustained by English gamekeepers as a result of the way they killed rabbits using their hands. Any of the MCP joint ulnar collateral ligaments could be injured in a fall if the mechanism of injury creates significant forces on the ligaments, but the 1st MCP joint is far more commonly injured than the others mentioned above.

An 18-year-old female comes to the emergency room stating she was raped just a few hours ago. She is not otherwise harmed, but does admit to a history of physical abuse between the ages of six and eight at the hands of a family member. Given this history, you know she is at higher than average risk for post-traumatic stress syndrome. You care for her immediate medical needs and are ready to release her. In counseling her on next steps, which of the following is most critical in order to improve her prognosis? A Begin therapy as soon as possible B Begin treatment with a serotonin uptake inhibitor C Begin treatment with a beta blocker to reduce symptoms D Begin treatment with prazosin to prevent sleep disturbance E Pursue legal procedures against the perpetrator as soon as possible

The Correct Answer is: A Therapy to aid in working through the traumatic experience (A), instituted as soon as possible after the event, has proven to be the most helpful way to avert or minimize post-traumatic stress disorder. This patient does not currently have any symptoms of PTSD, so pharmacologic treatment is not indicated. In a patient who is diagnosed with PTSD, SSRIs (B) may be helpful in reducing panic and improving sleep, beta-blockers (C) may reduce symptoms of anxiety, and prazosin (D) may help with sleep. Legal procedures (E) may help a patient in dealing with the event, but there are no data supporting this as a therapeutic intervention.

A 30-year-old man is brought to the emergency department by his wife for abdominal pain, nausea, vomiting, and diarrhea. She says he has been getting "worse and worse" for at least the past two months. He is "too tired" and his "muscles hurt too much" to play golf, formerly their favorite leisure activity. He has been depressed and reticent, instead of his usual talkative self. He won't eat much of anything, even when she cooks his favorite meals and she is sure he has lost considerable weight because his clothes "hang off him." He has refused medical evaluation until today, when the abdominal pain of approximately a week increased and he began having the vomiting and diarrhea. Initial impression is that the man is quite tanned, although it is the middle of winter. Closer examination reveals dark pigmentation in his skin folds and on the buccal mucosa. This presentation most strongly suggests which of the following? A adrenocortical insufficiency B diabetic ketoacidosis C hypercortisolism D hyperglycemic hyperosmolar state E hypothyroidism

The Correct Answer is: A This is a fairly classic presentation of adrenocortical insuffiency (Addison's disease). Diabetes mellitus, type 1, that has led to ketoacidosis (B) could also present with weight loss, nausea, and vomiting; but is also accompanied by hyperphagia and polydipsia, and does not have the hyperpigmentation associated with Addison's disease. Patients with hypercortisolism (Cushing Syndrome) (C) present with central obesity, hirsutism, thin skin, poor wound healing, and a host of other problems including emotional lability. Those with hyperglycemic hyperosmolar state (D) are typically older, have a high body mass index, and present with lethargy, confusion, and dehydration. Patients with hypothyroidism (E) usually have weight gain and constipation along with lethargy, fatigue, and weakness.

In the emergency department, you are asked to evaluate a 77-year-old man with a history of HTN who had a syncopal episode while chasing after his dog. He admits to recent episodes of chest discomfort, also associated with activity, as well as dyspnea at lower levels of activity including walking up one flight of stairs. On physical exam, a grade III/IV crescendo-decrescendo systolic ejection murmur can be heard best over the right upper sternal border. His EKG demonstrates NSR @ 80 bpm, with evidence of left ventricular hypertrophy. His troponin levels are negative for ischemia. What is the next most appropriate test or procedure? A Echocardiography B VQ scan C CT scan of the head D Serum D-dimer levels E MRI of the heart

The Correct Answer is: A This patient exhibits all the signs of progression of aortic stenosis, thus echocardiography is the next most appropriate test. A determination of severity can then be made, with possible cardiac catheterization if severe aortic stenosis is suspected, in preparation for surgical intervention if necessary. A VQ scan is appropriate if pulmonary embolism were suspected. A CT scan of the head could be considered if a head injury was suspected, but would not be the next step in the management of this patient. Serum D-dimer levels might be used to rule out pulmonary embolism, although it is a fairly nonspecific test. An MRI of the heart is not considered standard of care for aortic stenosis.

A 1-year-old boy is brought to the emergency department by his parents, who state that the child refuses to walk or crawl and begins crying when they stand him. Swelling to his right knee is noted; it is also warm to the touch and pain response is noted. His parents state that it seemed to start a couple of days ago and has gotten worse. They don't recall a trauma, but state that he seems to bruise easily. The child's mother states that she also bruises easily. Vitals are as follows: Temp: 38.0°C, HR: 70, RR: 15. What laboratory finding would you expect? A Prolonged aPTT (activated partial thromboplastin time) B Prolonged bleeding time C Prolonged PT (prothrombin time) D Prolonged thrombin clotting time E Thrombocytopenia

The Correct Answer is: A This patient has hemophilia A. Patients with severe hemophilia A have a prolonged aPTT; all of the other tests should be within the normal range.

A 48-year-old man presents to the emergency department with acute right upper quadrant tenderness, fever, and mild jaundice. Which of the following is most likely to be elevated in the blood? A bilirubin B creatinine C glucose D ketones E uric acid

The Correct Answer is: A This patient's signs and symptoms correlate with a suspected case of cholecystitis. Jaundice is associated with hyperbilirubinemia, in which the excess bilirubin can deposit in tissues such as the skin, sclera, and nails, causing a yellowish discoloration. Bilirubin is the waste product generated from the metabolism of hemoglobin.

A 62-year-old female with a known history of hypertension presents due to the abrupt onset of a severe headache, marked vertigo, nausea, vomiting, and ataxia. On physical exam, you note nystagmus, sensory loss, and weakness of the right face. What is the most likely diagnosis? A Cervical artery dissection B Cerebellar hemorrhage C Internal auditory artery occlusion D Lacunar infarction E Spinal cord compression

The Correct Answer is: B A cerebellar hemorrhage can result from multiple causes, similar to other intracerebral hemorrhages, such as hypertension, blood dyscrasias, trauma, and arteriovenous malformations. Patient presentation may range from an abrupt onset of headache to full coma, depending on the severity of the hemorrhage. Presentation is often similar to cerebellar infarction, with CT findings assisting in the differentiation. Symptoms may include all of those listed above as well as nuchal pain, altered consciousness, altered respiratory rate, abnormal eye movements, and impaired papillary responses. Cervical artery dissection may present with similar symptoms, including the abrupt onset of headache, but is not typically associated with ataxia, vertigo, or vomiting. Internal auditory artery occlusion is associated with vertigo and unilateral hearing loss. Lacunar infarction occurs within small resistance vasculature, with motor and sensory symptoms related to the impacted brain region. Spinal cord compression may be acute or progressive, with motor weakness and sensory loss caudad to the lesion.

A 61-year-old man arrives at the emergency department (ED) suffering an acute myocardial infarction as a result of coronary artery thrombosis. One of the agents administered to the patient is a thrombolytic agent. From the choices below, which drug is a thrombolytic agent? A abciximab B alteplase C warfarin D heparin E clopidogrel

The Correct Answer is: B Both warfarin and heparin are anticoagulants that are indicated for the prevention of thrombi. They do not actively lyse clots, but are capable of preventing further thrombogenesis. Both abciximab and clopidogrel are considered antiplatelet agents. Abciximab inhibits the activation of glycoprotein IIb/IIIa receptors on platelets, which helps to reduce platelet aggregation. Clopidogrel blocks adenosine diphosphate (ADP) receptors on platelets. The binding of ADP to these receptors is an important cellular mechanism in stimulating platelet aggregation. Alteplase converts plasminogen to plasmin, which then actively dissolves the fibrin threads associated with a thrombus.

A 68-year-old female presents to the emergency department due to exertional dyspnea and increased cough for the past week. The patient states that this has happened in the past, but she cannot recall which medication helped her. Physical exam reveals a sustained left ventricular impulse, an S3 gallop, and mild JVD. The chest x-ray is shown. What is the most likely diagnosis? A Acute respiratory distress syndrome B Congestive heart failure C Hypertrophic cardiomyopathy D Left pleural effusion E Right perihilar pneumonia

The Correct Answer is: B Congestive heart failure is common and increases in prevalence with age. Heart failure may be left sided or right sided, with patients often exhibiting signs of both as heart failure progresses. Patients will often present with shortness of breath, which may progress to increased dyspnea at rest and orthopnea. Physical exam findings may range from overt to subtle and may include jugular venous distention, adventitious lung sounds such as rales and rhonchi, hepatic enlargement, peripheral edema, and cardiac findings of a sustained left ventricular impulse, diminished first heard sound, S3 gallops, and an S4.

A 67-year-old female presents to the emergency department with abdominal pain, bloating, inability to pass gas or stool, and vomiting. Which of the following, if present, would be a contraindication to placing a nasogastric tube? A Esophagitis B Esophageal strictures C Esophageal varices D Peptic ulcer disease E Gastroesophageal refulx disease

The Correct Answer is: B Contraindications to nasogastric tube placement include choanal atresia, significant facial trauma, basilar skull fracture, esophageal stricture or atresia, esophageal burn, zenker's diverticulum, recent surgery on the esophagus or stomach, or a history of gastrectomy or bariatric surgery.

A 14-year-old boy presents to the emergency department with acute scrotal pain and vomiting for the past 2 hours. His left testicle is in extreme pain and he states the pain started while playing basketball in gym class. Which diagnostic test would help confirm your suspected diagnosis? A Transillumination B Doppler ultrasound C Urine cultures and sensitivity D Radionuclide imaging E Serum human chorionic gonadotropin levels

The Correct Answer is: B Doppler ultrasound (B) is the diagnostic imaging of choice to confirm testicular torsion, with radionuclide imaging (D) a distant second. Testicular torsion is time sensitive and MRI or more invasive imaging may lead to the delay of surgical intervention. Time is critical (< 6 hours) for the salvage of the affected testicle. Transillumination (A) is seen in hydrocele fluid. Urine cultures and sensitivity (C) can confirm urinary tract infections or sexually transmitted diseases causing epididymitis. If an intrascrotal tumor is suspected, serum tumor beta-human chorionic gonadotropin levels (E) should be obtained.

A 62-year-old man presents to the emergency department with aphasia and right lower extremity weakness that started about 4 hours ago. He now has progressing right upper extremity weakness, worsening right lower extremity weakness, and decreased sensation throughout his right side. This cerebral ischemia is best characterized as A transient ischemic attack B stroke in evolution C completed stroke D subarachnoid hemorrhage E global cerebral ischemia

The Correct Answer is: B During a stroke in evolution, symptoms will worsen or new symptoms will appear. A completed stroke is one in which neurologic symptoms have stabilized, whereas a transient ischemic attack produces deficits that resolve over time. This patient's symptoms do not match those of an acute subarachnoid hemorrhage. Global cerebral ischemia as seen in sudden cardiac arrest would involve loss of consciousness.

A 70-year-old man, with a history of HTN and aortic valve replacement 3 months ago, presents with complaints of arthralgia, myalgia, anorexia, fatigue, and weight loss over the last month, with recent dyspnea on exertion and lower extremity edema. Vital signs are as follows: Temperature 38°C, BP 102/64, P 98, RR 20. On physical exam, a new high-pitched, blowing, decrescendo diastolic murmur is noted along the left lower sternal border. Two separate blood cultures are positive for S. aureus, and found to be methicillin-resistant. A transesophageal echocardiogram demonstrates a paravalvular abscess. Which of the following is the most appropriate therapy in the management of this patient? A IV vancomycin B IV vancomycin, IV gentamicin, and PO rifampin with surgical treatment C IV amphotericin plus flycytosine, and surgical treatment D Outpatient IV ceftriaxone E IV penicillin G

The Correct Answer is: B In patients with prosthetic valve infection with methicillin-resistant S. aureus, the treatment of choice is IV vancomycin for 6 to 8 weeks, plus IV or IM gentamicin for the initial 2 weeks secondary to nephrotoxicity, and PO rifampin for 6 to 8 weeks, with susceptibility to gentamicin determined before initiation of rifampin. Surgical therapy decreases mortality in patients with S. aureus endocarditis, from over 70% with medical therapy alone to 25%, and should be considered in patients with paravalvular abscesses and symptoms suggestive of moderate to severe refractory congestive heart failure. Therefore, choice B is the most appropriate next step in the management of this patient. Choice A does not offer sufficient coverage for methicillin-resistant S. aureus. Choice C is appropriate therapy for infective endocarditis when the causative organism is Candida. Outpatient antibiotic therapy is only appropriate in patients who are stable, without clinical or echocardiographic findings to suggest complications, and IV Ceftriaxone is not appropriate for the treatment of methicillin-resistant S. aureus prosthetic valve endocarditis. Choice E is appropriate therapy for pencillin-susceptible streptococci, such as S. bovis.

A 66-year-old woman presents to the emergency department with a complaint of abdominal pain and distension for the past 3 days. Examination reveals a protuberant abdomen with diminished bowel sounds and tympany to percussion. Flat and upright abdominal radiographs reveal distended loops of bowel with prominent haustral markings. Which of the following etiologies is the most likely cause of the patient's condition? A Volvulus B Adenocarcinoma C Diverticular disease D Strangulated hernia E Adhesions

The Correct Answer is: B Large bowel obstructions are most commonly caused by an adenocarcinoma (65%). This is followed in decreasing incidence by diverticular scarring and volvulus. Adhesions are the most common cause of small-bowel obstruction but are rare as a cause of large bowel obstruction. The presence of haustral markings on radiographic evaluation helps differentiate between small and large bowel involvement.

A 22-year-old patient with sickle cell disease presents to the emergency department complaining of chest pain, fever, and non-productive cough. On physical exam his temperature is 100.6˚F, BP is 144/88, pulse is 110, respiratory rate is 24, and pulse oximetry is 84%. CBC shows a WBC of 11,500, hemoglobin of 8.3%, and hematocrit of 28%. What is the most likely diagnosis? A Acute bronchitis B Acute chest syndrome C Asthma D Bronchiectasis E Pneumothorax

The Correct Answer is: B Patients with sickle cell disease are prone to acute chest syndrome, resulting from sickling of cells within the lung that typically presents with chest pain, tachypnea, cough, fever, and oxygen desaturation. Acute bronchitis (A) and bronchiectasis (D) typically present with a productive cough without significant oxygen desaturation and anemia. Patients with asthma (C) and spontaneous pneumothorax (E) will be afebrile and have normal hemoglobin and hematocrit.

Which of the following is the initial treatment step in an adolescent who presents to the emergency department with status epilepticus? A IV glucose B stabilize airway C arterial blood gas D IV diazepam therapy

The Correct Answer is: B Status epilepticus is a medical emergency and is defined as seizure activity that lasts a minimum of 30 minutes. This results in hypoxia, acidosis, cerebral edema, and structural damage. In addition, fever, respiratory depression, hypotension, and death may occur. There are both convulsive and nonconvulsive types of status epilepticus. Because of its emergency status and potential complications, the clinician needs to initiate the ABCs (airway, breathing, circulation). Therefore, the first line of treatment is to establish and maintain an airway, oxygen is next, and then circulation, which encompasses pulse, blood pressure, and IV access. Once the IV is established, the orders should be for administering glucose-containing fluids and IV drug therapy with diazepam, lorazepam, or midazolam as well as administer phenytoin and phenobarbital. Arterial blood gases should be ordered and any abnormalities should be corrected appropriately. Finally, the clinician should determine the underlying cause: trauma, structural disorder, infection, lactic acidosis, toxins, and uremia. Maintenance drug therapy is necessary until the underlying cause is determined and rectified.

A 63 year-old woman presents to the clinic complaining of increasing dyspnea over the last two weeks. Which of the following diagnoses is supported by the chest x-ray below? A Dissecting aortic aneurysm B Pleural Effusion C Pneumonia D Pneumothorax E Pulmonary Embolism

The Correct Answer is: B The patient has a left-sided pleural effusion (B). The mediastinum is not widened as seen in dissection aortic aneurysms (A) and although the lung markings are absent in the left lower lobe region this is due to the accumulation of fluid (B) and not air as seen in pneumothorax (D). Clear signs of pneumonia (D) or pulmonary embolism (E) aren't present on the chest x-ray, but are potential underlying causes of effusion.

A 19-year-old woman presents to the emergency department complaining of headache. The headaches are generalized and increasing in intensity. They have not responded to over-the-counter (OTC) medications. She complains of approximately 1 week of blurred vision, intermittent diplopia, and vague dizziness. Her medical history includes obesity and acne. She takes Accutane and oral contraceptives. She is found to have bilateral papilledema, visual acuity of 20/30 on physical examination, and a normal MRI of the brain. The next most appropriate step would be A CT scan of the head B lumbar puncture C therapy with high-dose prednisone D stat cerebral arteriogram E reassurance and follow-up in the office in 6 months

The Correct Answer is: B The presence of headache associated with papilledema raises the concern for a brain tumor. The MRI excluded a mass lesion, raising a strong suspicion of pseudotumor cerebri. This is also known as benign intracranial hypertension. It is not a benign condition, however, since it causes severe headache and may result in visual loss. It is particularly frequent in obese adolescent girls and young women. The etiology is unknown but may be associated with the use of oral contraceptives, vitamin A, and tetracycline. The presentation consists of headaches caused by an increase in intracranial pressure and blurring of vision. There may be diplopia, but the remainder of the neurologic examination is unremarkable. Papilledema is virtually always part of the presentation. The mental status is normal. The differential diagnosis includes venous sinus thrombosis, sarcoidosis, and tuberculosis or carcinomatous meningitis. The last two are excluded by lumbar puncture. An abnormal cerebrospinal fluid is not consistent with pseudotumor cerebri. The diagnosis is made by excluding mass lesions with CT scan or MRI and demonstrating markedly increased intracranial pressure by lumbar puncture. The treatment involves weight loss, diuretics, and steroids. Repeat lumbar punctures to remove cerebrospinal fluid and decrease intracranial pressure are effective. In cases that are unresponsive to these measures, lumbar-peritoneal shunting is effective, as is unilateral optic nerve sheath fenestration. Effective treatment can improve headaches and prevent vision loss.

A 27-year-old African American female presents to the emergency department with low blood pressure of 100/40, palpitations, and shortness of breath. She is currently under treatment for Wolff-Parkinson-White syndrome and has been taking procainamide for the last two years. An electrocardiogram is obtained on the monitor and reveals the rhythm strip shown. What is the treatment of choice for this patient? A Intravenous calcium B Intravenous magnesium C Oral potassium D Subcutaneous epinephrine E Metoprolol

The Correct Answer is: B The rhythm strip reveals ventricular tachycardia in the form of torsades de pointes. In this case, the primary medical intervention is to administer magnesium sulfate to counter the irregular activity. Antiarrhythmics, antidepressants, and some antibiotics can be responsible for this arrhythmia. In addition to the magnesium, administration of beta-blockers can also be helpful.

An 89-year-old female was found to have had an episode of syncope while at home. There was no history prior to the event, and the patient denies any chest pain prior to the episode occurring. On exam, the patient is alert, awake, and oriented, and is only complaining of mild shortness of breath. Her ECG is as shown. Which of the following is the best choice for treating this patient? A No treatment B Permanent pacemaker C Digitalis therapy D Ablation therapy E Beta blockers

The Correct Answer is: B This patient has a third-degree AV block that is symptomatic, with syncope and mild shortness of breath. The treatment for this patient is pacemaker insertion. Immediate insertion depends on the vital signs of the patient. If the patient's vital signs are stable, a transdermal pacer can be used until a scheduled pacer insertion within 24 hours. If the vital signs are not stable, immediate intervention with either central venous pacing wires or an immediate pacer insertion is warranted.

A 66-year-old male with a history of hypertension, diabetes mellitus, and hypercholesterolemia presents by emergency medical services (EMS) to the emergency department complaining of severe chest pain with radiation into his back. The patient states that he was feeling well in the morning, but while performing some light activity he felt a "ripping" sensation in his back, which he initially thought was a pulled muscle. The pain continued and the patient started to have chest pain, shortness of breath, and lightheadedness. On initial examination the patient is still in pain, pale, diaphoretic, and has a blood pressure of 85/40. His chest is clear to auscultation, and he has a 3/6 diastolic murmur best appreciated at the base of the heart. Given this clinical scenario, what is the most likely diagnosis? A Pneumothorax B Dissecting thoracic aneurysm C Acute myocardial infarction D Pulmonary embolus E Esophageal perforation

The Correct Answer is: B This patient is exhibiting a history and physical examination that is consistent with a thoracic aneurysm. The patient's history of hypertension, along with the "ripping" sensation in his back and hypotension give a clinical presentation that is suggestive of a thoracic aneurysm dissection (B). A pneumothorax (A) would have more pleuritic characteristics, and chest pains without the ripping sensation or loud diastolic murmur would be more likely in a situation of myocardial infarction (C). Pulmonary embolus (D) and esophageal perforation (E) would typically not present with the above complaints or physical exam findings.

A 64-year-old man has been experiencing signs and symptoms compatible with diverticular disease for the past 3 weeks. He now presents to the emergency department malnourished with severe left-sided lower abdominal pain. After appropriate workup and hydration, he is taken to the operating room where a perforated sigmoid colon is discovered with gross contamination. What is the most appropriate surgical intervention at this time? A Left colectomy with primary anastomosis B Hartmann procedure C Proctocolectomy D Abdominoperineal resection E Low anterior resection

The Correct Answer is: B This vignette is consistent with an emergent resection in an unprepared patient. The most appropriate therapy for an acute perforation is a Hartmann procedure, which includes resection of the affected portion of the bowel, a temporary diverting colostomy, and oversewing of the distal rectal stump; the second stage of the procedure will involve taking down the colostomy with anastomosis to the rectal stump. A colectomy with a primary anastomosis should not be done when the bowel is unprepared due to the significant risk of infection and leakage of the bowel at the site of the anastomosis. Abdominoperineal resection is used in the treatment of malignant disease of the lower rectum. In this procedure, a permanent colostomy is created and the entire rectum, anal canal, and anus are removed. In the management of benign disease of the lower rectum, a proctocolectomy is appropriate to preserve anal function.

A 78-year-old woman with known diabetes mellitus type 2 is brought to the emergency room after a neighbor became concerned when newspapers began piling up on her doorstep and called the police. In the emergency department, she is found to be lethargic and disoriented, with tenting of the skin, sunken eyes, and dry mucous membranes. She is hypotensive and has a rapid pulse. She is wearing a diaper that apparently has been in place for several days, but is barely moist. What is the most appropriate fluid therapy for them to initiate? A 0.45% saline B 0.9% saline C 5% dextrose in water D 5% dextrose in 0.45% saline E Lactated Ringers

The Correct Answer is: B This woman is severely dehydrated so 0.9% saline is indicated. If she were less dehydrated, 0.45% saline (A) would be appropriate because of likely hyperosmolality. Once her glucose has dropped to 250 mg/dL, she should receive dextrose in water (C) or 0.45% saline (D) to prevent her glucose level from dropping too low. Lactated Ringers ı is contraindicated in patients who are likely to have severe acidosis or alkalosis.

A 46-year-old African American male is seen in the emergency department with upper right quadrant pain that radiates to the right infrascapular area. The pain is colicky and was precipitated by a meal of fried fish and French fries. Which of the following diagnostic studies is the initial study of choice for this patient? A Plain abdominal x-ray B Ultrasonography C Radionuclide scan (HIDA scan) D Computed tomography (CT)

The Correct Answer is: B Ultrasonography is the first-line study in the evaluation of patients presenting with signs and symptoms of biliary disease. The sensitivity and specificity is 95%. It can detect stones, dilation of biliary ducts, thickening of the gallbladder, and pericolic collections of fluid and can also provide information pertaining to associated liver or pancreatic pathology.

You are performing a mental status exam on a 19-year-old male patient who was brought to the Emergency Department because of bizarre behavior. As you are speaking with him he keeps repeating words that rhyme with words that either you or he say, but make no sense and are unrelated to your questions. How will you document this behavior in his chart? A Circumstantiality B Clanging C Flight of ideas D Perseveration E Tangentiality

The Correct Answer is: B Word association based on rhyme is called clang association (B) and may be seen in psychotic disorders. Circumstantiality (A) is a disturbance in fluency where the speaker meanders on many side topics before returning to the topic at hand. Flight of ideas (C) is a rapid transition from thought to thought, leading to the speaker losing track of the original idea. Perseveration (D) is a fixed focus returning again and again to the same thought. Tangentiality (E) refers to a disturbance in continuity where the speaker shifts from one thought to another that may be only vaguely related.

A 7-year-old male presents to the emergency department with right leg pain after falling off a swing at the school playground. Imaging of the right lower extremity shows the following fracture pattern: Which type of Salter-Harris Classification is observed? A Type I B Type II C Type III D Type IV E Type V

The Correct Answer is: C A Salter-Harris Type III fracture (C) involves a portion of the epiphysis only. Type I (A) is the entire epiphysis, Type II (B) is the entire epiphysis along with a portion of the metaphysis, Type IV (D) involves a portion of the epiphysis along with a portion of the metaphysis, and Type V (E) is a compression injury of the epiphyseal plate (nothing is "broken off").

A 24-year-old intoxicated male presents to the emergency department after being in a fight. He was punched in the nose, and now has mild deformity of the nose and some epistaxis. An x-ray reveals a fractured nasal bone. During his physical exam, what must you look for in order to prevent permanent destruction of his nasal septum? A Orbital fracture B Posterior epistaxis C Septal hematoma D Facial fracture E Deviated septum

The Correct Answer is: C A septal hematoma can cause ischemic necrosis of the nasal septal cartilage if not identified and drained. A deviated septum can be expected with a nasal bone fracture, and must be addressed by the otolaryngologist. Excessive epistaxis that does not resolve with direct pressure and anterior packing may indicate a posterior bleed.

A 68-year-old female presents to the emergency department with signs and symptoms of an acute ischemic stroke. The initial CT scan is normal. Her blood pressure is 164/105. What is the most appropriate treatment for the blood pressure of this patient? A Atenolol PO B Clonidine PO C Close monitoring D Labetolol IV E Nicardipine IV

The Correct Answer is: C Aggressively lowering blood pressure may decrease blood flow to the ischemic tissue, thus decreasing the chances of recovery or increasing the risk of further infarction. In the setting of an acute ischemic stroke, blood pressure elevation should be monitored closely, with some elevation expected. This elevation is expected to decline without medication in the first few hours to days, but if elevation continues to a systolic blood pressure greater than 220mmHg, or mean arterial pressure greater than 120mmHg, medication is advised. Medications may include intravenous labetolol or nicardipine, with close monitoring of the patient. After the acute phase following a stroke, appropriate oral medications may be considered for outpatient hypertension management.

A 16-year-old girl is brought to the emergency department by ambulance after reportedly ingesting "a bottle of aspirin." Vital signs are temperature 37.8°C oral; pulse 94/min; respirations 30/min; blood pressure 100/68 mm Hg. What would you expect the blood gases to show that would confirm she had swallowed the aspirin? A anion gap metabolic acidosis with respiratory acidosis B nonanion gap metabolic acidosis with respiratory alkalosis C anion gap metabolic acidosis with respiratory alkalosis D nonanion gap metabolic acidosis with respiratory acidosis

The Correct Answer is: C An acute salicylate overdose (greater than 150 mg/kg) will produce symptoms of salicylate intoxication. Chronic salicylate intoxication occurs with ingestion of greater than 100 mg/kg/day for at least 2 days. Salicylates affect most organ systems, leading to various metabolic abnormalities. Because salicylates are a gastric irritant, symptoms of vomiting and diarrhea occur soon after the overdose, which may contribute to the development of dehydration. Salicylates stimulate the respiratory center leading to hyperventilation and hyperpnea resulting in respiratory alkalosis and compensatory alkaluria. A characteristic feature of salicylate intoxication is the coexistence of a respiratory alkalosis with a widened anion gap metabolic acidosis.

A 33-year-old IV drug user presents to the emergency department with chills, diaphoresis, anorexia, and malaise. On physical exam, her temperature is 40°C, BP 98/55, P 115 bpm, and RR 22. Two separate blood cultures are positive for S.aureus. Which of the following physical exam findings would confirm a clinical diagnosis of infective endocarditis, according to the Duke criteria? A Increase in valvular regurgitation B Irregularly irregularly pulse C Osler's nodes D Buccal hemmorhages E Koplik spots

The Correct Answer is: C Choice C, Osler's nodes, confirms the clinical diagnosis of infective endocarditis, as it is a minor criteria. The Duke criteria for the clinical diagnosis of infective endocarditis requires the documentation of two major criteria, or one major criteria and three minor criteria, or five minor criteria. The patient demonstrates the presence of one major criteria (two separate blood cultures with typical microorganisms for infective endocarditis) and two minor criteria (fever greater than 38.0°C and predisposing condition of IV drug use). Only a new valvular regurgitation, not an increase or change in preexisting murmur, is considered sufficient to qualify as a major criteria, so choice A is incorrect. An irregularly irregular pulse, choice B, is commonly seen in patients with atrial fibrillation, not with infective endocarditis. Choice D, conjunctival hemorrhages, not buccal hemorrhages, are one of the minor criteria. Choice E, Koplik spots, are buccal lesions seen in patients infected with measles, whereas the presence of Roth's spots does fulfill one of the minor criteria

A 55-year-old woman with a history of emphysema, who is undergoing chemotherapy for lung cancer, comes to the emergency department complaining of a sudden increase in dyspnea, with exertion and fatigue. On physical exam, hypotension, pulsus paradoxus, and muffled heart sounds are noted. On transthoracic echocardiography, cardiac tamponade is noted, with over 200 mL of pericardial fluid described. Which of the following would be this patient's most likely electrocardiographic finding (Figure 7)? A Torsades de pointes B U waves C Electrical alternans with sinus tachycardia D Peaked T waves E Convex elevation of the J point

The Correct Answer is: C Choice C, electrical alternans with sinus tachycardia, a beat-to-beat alteration in one or more components of the ECG signal, is considered a specific sign of pericardial effusion, often with cardiac tamponade, as it represents the periodic swinging motion of the heart in the effusion at a frequency that is ½ the heart rate. Choice A, torsades de pointes, is a type of ventricular tachycardia frequently seen, and is associated with electrolyte disturbances or the use of certain types of antiarrhythmic drugs. Choice B, U waves, are associated with hypokalemia. Choice D is frequently noted with severe hyperkalemia. Choice E, convex elevation of the J point, is seen in patients suffering from hypothermia.

A 20-month-old boy is brought into the emergency department by his parents. They state he has not been feeling well for 2 days and this morning noted he was "shaking all over" and was not responding to commands. This went on for less than 10 minutes and has never happened before. His current rectal temperature is 100.7°F. The seizures are characteristic of A absence seizures B Lennox-Gastaut syndrome C febrile seizures D infantile spasms E juvenile myoclonic epilepsy

The Correct Answer is: C Febrile seizures can occur in children younger than 5 years when accompanied by a fever. They are characterized by a brief generalized motor seizure. Absence seizures are generalized seizures characterized by a loss of consciousness without motor involvement, typically seen in older children. Lenox-Gastaut syndrome presents in childhood as well but is usually associated with developmental delay and seizures of akinetic and myoclonic nature (referred to as drop attacks). Infantile spasms occur without relation to systemic illness and are massive myoclonic events with bending at the waist. Juvenile myoclonic epilepsy evolves in the teenage years and is characterized by repeated episodes of myoclonic seizure activity.

A 16-year-old high school boy presents to the emergency department 4 hours after sustaining an abrasion to his knee after a fall while rollerblading on the school playground. His school immunization record reveals that his last diphtheria, tetanus, and pertussis (DTaP) booster was administered at age 4. In this situation, which of the following is the MOST appropriate plan? A administer tetanus toxoid B administer adult tetanus and diphtheria toxoid (Td) C administer diphtheria, tetanus toxoid, and acellular pertussis (Tdap) vaccine D administer tetanus immune globulin

The Correct Answer is: C Generalized tetanus (lockjaw) is a neurologic disease caused by Clostridium tetani. Although any open wound is a potential source for contamination with C tetani, those with dirt, soil, feces, or saliva are at increased risk. Tetanus-prone wounds contain devitalized tissue, especially those caused by punctures, frostbite, crush injury, or burns. Recommendations for tetanus prophylaxis in a child with a laceration or abrasion depend upon the number of previous vaccinations, occurrence of last booster, type of wound (clean or tetanus-prone), and age of child. In this case, the patient is older than 7 years and had all of his previous immunizations; however, his most recent booster was greater than 10 years ago. Thus, he should receive an adult-type diphtheria and tetanus toxoid with acellular pertussis. In most cases, when tetanus toxoid is required for wound prophylaxis in a child older than 7 years, the Td instead of tetanus toxoid alone is recommended so that diphtheria immunity is maintained. If tetanus immunization is not up to date at the time of wound treatment, then the immunization series should be completed according to the primary immunization schedule. If a child is younger than 7 years, then the diphtheria, tetanus, acellular pertussis (DTaP) booster is indicated, unless there is a contraindication for pertussis, in which case the diphtheria and tetanus (DT) booster should be administered. Tetanus immune globulin (TIG) is recommended for treatment of tetanus. Under special circumstances, a patient infected with the human immunodeficiency virus (HIV) with a tetanus-prone wound should also receive TIG in addition to the prophylactic vaccine.

A 15-year-old boy suddenly collapses on the basketball court; his sports physical conducted at the beginning of the year did not elicit any abnormal findings. Basic life support initiated at the scene, however, is unsuccessful in resuscitation. Which of the following is the most likely etiology of his sudden death? A mitral valve prolapse B surgically corrected aortic stenosis C hypertrophic cardiomyopathy D rheumatic heart disease

The Correct Answer is: C Hypertrophic cardiomyopathy in adolescence is typically due to familial hypertrophic cardiomyopathy with an incidence of 1:500. Many patients are asymptomatic until a sporting event, which may cause symptoms, specifically sudden cardiac death. Examination may demonstrate a palpable or audible S 4 , an LV (left ventricular) heave, systolic ejection murmur (may need to stimulate cardiac activity), and/or a left precordial bulge. Echocardiography is the gold standard for diagnosis but family history should be assessed. Stress testing is indicated to assess for ischemia and arrhythmias. Strenuous activities are prohibited for these patients. The other cardiomyopathies (dilated and restrictive) are next but are not as common. Congenital structural abnormalities of the coronary arteries are the next most common cause. Valvular disorders, including surgically repaired aortic stenosis, are typically not causes of sudden death, but these patients should be screened for symptoms and stress tested as necessary.

An 18-year-old woman is transferred to your emergency department from a local college infirmary. She presented yesterday with a complaint of headache but became confused and is now febrile. You notice a petechial rash on physical examination and her cerebrospinal fluid comes back with increased WBCs, increased protein, and decreased glucose. What is the most likely organism responsible for her meningitis? A Haemophilus influenzae B cytomegalovirus C Neisseria meningitidis D Mycobacterium tuberculosis E coxsackievirus B

The Correct Answer is: C Neisseria meningitidis and Streptococcus pneumoniae are the most common etiologic agents for bacterial meningitis in this patient's age group. So much so that many colleges and universities require a vaccine for students who live in dormitories. Her fever and the cerebrospinal fluid values are consistent with a bacterial and not a viral infectious source for the meningeal irritation.

A 33-year-old woman treated with trifluoperazine for the past 3 months is seen in the emergency department because of recent-onset fever, stiffness and tremor, as reported by her accompanying sister. The patient also appears to be mildly confused when asked about location, day, and time. Her temperature is 104.5°F, and her serum creatine kinase (CK) level is markedly elevated. Which of the following has most likely occurred? A a delayed allergic reaction has occurred with trifluoperazine B tardive dyskinesia has begun to develop in the patient C the patient has developed neuroleptic malignant syndrome D the patient has developed serotonin syndrome E the patient has overdosed on trifluoperazine

The Correct Answer is: C Neuroleptic malignant syndrome is an uncommon but serious complication with therapeutic doses of antipsychotic drug therapy, particularly the first-generation (typical) class. Cardinal signs and symptoms include a body temperature above 100.4°F, altered state of consciousness, autonomic dysfunction, and rigidity.

A 21 year-old female presents to the emergency department after having a cast applied to her right arm earlier that day. Approximately one hour ago she began having extreme 10/10 pain in her right arm and is in visible distress. When considering the diagnosis of compartment syndrome, permanent damage to the muscle begins after how many hours of ischemia? A >2 hours B >6 hours C >8 hours D >12 hours E >24 hours

The Correct Answer is: C Permanent damage results after >8 hours (C) of ischemia. Nerves begin to lose conduction within 2 hours of onset of elevated pressures. Neurapraxia can occur within 4 hours, and irreversible damage occurs 8 hours after elevated pressures. Functional impairment is unlikely when compartment syndrome is diagnosed and treated within 6 hours of its onset. While deficits can occur prior to 8 hours, permanent damage is usually not seen before 8 hours (A and B). After 12 or 24 hours, permanent damage has already resulted in most cases (D and E). The key to early detection of compartment syndrome is a high index of clinical suspicion.

A 4 year-old male presents to the emergency department with vomiting, severe abdominal pain, and diarrhea that occurred 2 hours after "getting candy from Grandma's purse". Which of the following medications did the child most likely ingest? A Cisplatin B Erythropoietin C Ferrous sulfate D Lisinopril E Vincristine

The Correct Answer is: C Stage 1 of iron toxicity secondary to ferrous sulfate ingestion (C) is characterized by acute GI irritation; this is followed by a latent phase (Stage 2) that can then progress to systemic iron toxicity (Stage 3), that can progress to hepatic failure (Stage 4) or delayed sequelae (Stage 5). Cisplatin (A), erythropoietin (B), and vincristine (E) are parenterally administered and unlikely to be present in a patient's purse. Lisinopril (D) toxicity consists of cardiovascular symptomatology (e.g., hypotension and tachycardia).

A 27-year-old African American with sickle cell anemia presents to the emergency department with acute onset intractable pain. She is taking quick, shallow breaths and her oxygen saturation is 84% on room air. She appears desiccated, states she hasn't eaten in the last 24 hours, and says that she "just doesn't feel well." She is also afebrile. What should your next course of action be? A Start morphine, hydrate, and start antibiotics B Start morphine, oxygen, and start antibiotics C Start oxygen, hydrate, and exchange transfusion D Start oxygen, hydrate, and give pneumococcal vaccination E Start oxygen, hydrate, and start antibiotics

The Correct Answer is: C Start oxygen, hydrate, and exchange transfusion Exchange transfusions are primarily indicated for the treatment of intractable pain crises, priapism, and stroke. Patients should be kept well hydrated, and oxygen should be given if the patient is hypoxic. Antibiotics would be used if there was an infection identified, but are not part of initial treatment in a sickle crises.

A 14-year-old boy presents to the emergency department with acute scrotal pain and vomiting for the past 2 hours. His left testicle is in extreme pain and he states the pain started while playing basketball in gym class. Which diagnosis is highest on your differential in regards to his clinical presentation? A Epididymitis B Hydrocele C Testicular torsion D Varicocele E Intrascrotal tumor

The Correct Answer is: C Testicular torsion (C) is most common between ages 12-18 with the classic presentation of abrupt and severe onset of pain with nausea/vomiting. The most common cause of painless scrotal swelling in children is a hydrocele (B). Epididymitis (A) usually has an insidious onset, commonly presenting with urinary frequency, dysuria, or fever. Varicoceles (D) and intrascrotal tumors (E) usually present as painless scrotal edema.

What is the initial treatment for a patient who is having an acute episode of supraventricular tachycardia? A Caffeine B Beta blockers C Valsava maneuver D No treatment E Synchronized cardioversion

The Correct Answer is: C The initial treatment that should be attempted is a simple vagal maneuver to break the reentry tachycardia. If this is unsuccessful, pharmacologic therapy is indicated.

A 21-year-old man presents to the emergency department complaining of a swollen left ear (see image) after he experienced blunt trauma in a collegiate wrestling match. Which of the following is the most appropriate management for this condition? A Evacuation B Evacuation followed by antibiotics C Evacuation with subsequent antibiotics and splinting D Ice and compression E Reevaluation in 24 hours

The Correct Answer is: C The patient has an auricular hematoma that must be evacuated to prevent cartilage necrosis or infection. Evacuation, antibiotics, and splinting (C) remove the hematoma, minimize the risk of infection, and help prevent the re-accumulation of blood.

A 17-year-old boy high school wrestler is brought into the emergency department after he collapsed at a wrestling match. He spent time fully clothed in a hot sauna prior to the match to try to "make weight." Labs are ordered, and results come back as follows: Which IV fluid regimen would most effectively treat this patient's hypernatremia? A quarter normal (hypotonic) saline B half-normal saline C isotonic (normal) saline D dextrose 5% in water E lactated Ringer's

The Correct Answer is: C The patient presents with a combination of inadequate fluid intake and excessive losses due to perspiration, resulting in hypovolemia and hypernatremia. The most common causes of hypernatremia are inadequate fluid intake resulting in hemoconcentration and diabetes insipidus (DI), resulting in excessive renal fluid losses. Normal urine osmolality is 500 to 850 mOsm/kg but can range from 50 to 1,200 mOsm/kg depending on the patient's fluid intake. Urine osmolality >400 mOsm/kg indicates that the renal fluid-conserving mechanism is intact, as the kidneys are working to preserve volume. A lower urine osmolality would be consistent with DI, characterized by a lack of response to anti-diuretic hormone (ADH), resulting in excessive urinary losses of water with worsening hypernatremia. Treatment is directed at the cause. If the patient is dehydrated, restoring fluid volume is the goal. If the patient has DI, treating the underlying disease will lower the serum sodium level. For this dehydrated patient, the treatment would be to administer isotonic (normal) saline, which contains 0.9% sodium, because of the large free water deficit. Quarter-normal saline contains 0.25% sodium, half-normal saline contains 0.45% sodium, and lactated Ringer's solution is similar to half-normal saline in its sodium content. Dextrose 5% in water (D5W) contains no electrolytes. Isotonic saline is the appropriate choice because it treats not only the volume deficit but the serum osmolality as well. Its osmolality (308 mOsm/kg) is often lower than the plasma osmolality because of the hypovolemic state and, therefore, helps restore normal serum osmolality. Once serum osmolality becomes more normal, the isotonic saline can be replaced by D5W to replace the remaining free water deficit. If the free water deficit were less dramatic, initial IV fluid treatment could be half-normal saline, followed by D5W.

A 43-year-old male presents to the Emergency Department complaining of right eye pain after treating his yard with fertilizer and lime. He attempted to flush his eye at home without relief of pain. Which of the following is the most appropriate initial step in managing this patient's symptoms? A Double evert his eyelids to look for remaining foreign bodies B Fluorescein stain his eye C Instill proparacaine 0.5% ophthalmic solution D Irrigate his eye until the pH is between 6.8 and 7.4 E Refer to ophthalomogist

The Correct Answer is: C The patient requires all of the above steps and should be given pain relief (C) prior to thoroughly flushing the eye (D), removing foreign bodies (A), assessing for corneal injuries (B), and referring to ophthalmology (E).

A 19 year-old-male is brought to the emergency department due to psychosis. During the evaluation, he is noted to appear unkempt and has reduced motor activity. His affect is flattened, and he is using made-up words and phrases during speech. This patient is exhibiting what type of verbal utterance? A Echolalia B Hallucination C Neologism D Catatonia E Verbigeration

The Correct Answer is: C The use of made-up words or phrases is termed neologism. Echolalia is a repetition of words spoken by others. Hallucinations are perceptual distortions, and may be auditory or visual. Catatonia describes a state in which a person is withdrawn and shows minimal bodily movement, or one in which there is severe excitement and the person shows purposeless and stereotyped movements. Verbigeration is the repetition of senseless phrases or words.

A 62-year-old man with a history of hypertension, diabetes mellitus type 2, hyperlipidemia, and chronic tobacco use presents to the emergency department with complaints of retrosternal chest pressure associated with diaphoresis, nausea, and dyspnea, radiating down his left arm for the last 45 minutes after mowing his lawn. His EKG is shown (Figure 3). Which of the following is the most likely diagnosis given this patient's history and EKG findings? A Acute pulmonary embolism B Esophageal spasm C Acute myocardial infarction D Costochondritis E Aortic dissection

The Correct Answer is: C This patient is demonstrating evidence of acute anterolateral myocardial infarction on EKG, with ST segment elevation across the precordial leads, and reciprocal changes in the inferior leads, indicative of left anterior descending coronary artery stenosis. In addition, this patient's history increases the risk of acute myocardial infarction; therefore, choice C is the most likely diagnosis. Acute pulmonary embolism may also present with chest discomfort, and the patient's history of chronic tobacco use does increase this patient's risk of a hypercoaguable state. However, EKG changes with pulmonary embolism differ from those pictured, in that usually on an EKG, a pulmonary embolism presents with a prominent S wave in lead I, Q wave in lead III, and inverted T wave in III (S1/Q3/T3). Choices B, D, and E may also present with retrosternal chest discomfort; however, no EKG changes would be noted.

A 17-year-old male is brought to your Emergency Department by his girlfriend. She states that he has been behaving strangely for the last three days, with rapidly fluctuating moods ranging from euphoric to irritable and paranoid. The patient states that he is fine, just a little nervous about an upcoming test in school. His pulse is 126 beats per minute, BP 182/106, pupils are widely dilated, and he is diaphoretic. What is his most likely diagnosis? A Acute anxiety B Bipolar disorder C Cocaine intoxication D Heroin intoxication E Thyrotoxicosis

The Correct Answer is: C This patient's presentation with tachycardia, hypertension, diaphoresis, and mydriasis along with the behavioral changes is consistent with cocaine intoxication (C). His physical symptoms could be explained by thyrotoxicosis (E), but he has no history of hyperthyroid symptoms such as weight loss and it would not explain the mydriasis. Acute anxiety (A) or bipolar disorder (B) might explain some of the mood changes but would not account for all of his physical signs. Heroin withdrawal (D) would cause some similar symptoms, but intoxication would cause somnolence and pinpoint pupils.

An 80-year-old male nursing home patient is brought to the emergency department with abdominal distension. A plain film of the abdomen is pictured below. Which of the following is the most likely diagnosis? A Small bowel obstruction B Cecal volvulus C Sigmoid volvulus D Toxic megacolon

The Correct Answer is: C A volvulus is an obstruction of the colon due to a loop of bowel that has rotated more than 180 degrees on its axis with the mesentery. The most common site for a volvulus is the sigmoid colon (65%). A sigmoid volvulus is associated with abdominal pain and distension. Plain films of the abdomen would show a characteristic "bent inner tube" appearance. Sigmoidoscopy can be used to decompress the bowel by gently releasing the area of obstruction. Following decompression, a rectal tube is inserted to act as a stent to prevent the bowel from twisting upon itself again.

An 8-year-old girl is rushed to the emergency department by her parents because she has become delirious. The child was diagnosed with influenza three days prior. Her parents say that she had begun vomiting yesterday, almost nonstop, and has not been able to hold down fluids. They also note that she has been breathing rapidly. Your exam reveals a tachypneic, disoriented female with hyperreflexia, a positive babinski reflex, and liver enlargement. CSF analysis reveals a normal protein and cell count. What is the most likely diagnosis? A Bacterial meningitis B Guillain Barre syndrome C Measles encephalitis D Reye's syndrome E Viral meningitis

The Correct Answer is: D Although rare, Reye's syndrome is associated with viral infections, salicylate use during illness, and metabolic disorders. Illness is associated with liver fat deposition and degeneration, intractable vomiting, and mental status changes, which may progress to seizures, delirium, and coma. Cerebral edema contributes to these changes and other neurologic findings. Meningeal signs are more consistent with meningitis. Measles encephalitis typically presents days to weeks after the pathognomic measles exanthem and clinical findings. Guillain Barre has been associated with influenza infection, and signs and symptoms would include evolving weakness with ascending paralysis and extremity dysesthesias.

A 24-year-old HIV-positive man comes to the emergency department complaining of severe left-sided chest discomfort, which radiates through to the left trapezius region. On coming into the room, you note that he is sitting up and hunched forward. Prior to examining him, you have reviewed his chart. Laboratory findings demonstrate troponins x 3, which are negative for myocardial ischemia. His EKG demonstrates diffuse ST segment elevations throughout. Which of the following physical exam findings would be most likely in this patient? A Roth spots B Splenic enlargement C Janeway lesions D Pericardial friction rub E Splinter hemorrhages

The Correct Answer is: D Choice D is the most likely finding, as this patient is exhibiting signs, symptoms, and EKG findings pathognomonic for acute pericarditis, which is likely infectious in the setting of a patient with HIV. A pericardial friction rub is heard best with the patient in a seated position, during expiration, and is frequently found in patients with pericarditis. Choice A B, C, and E are physical exam findings seen in acute bacterial endocarditis.

During a hospitalization for pneumonia, troponin levels are drawn on a 62-year old-man with a history of hypertension, hyperlipidemia, and chronic tobacco use, and found to be elevated above the 99 th percentile of normal. If acute myocardial infarction is ruled out, which of the following disease entities could also cause troponin elevation? A Mitral regurgitation B Gout C Parkinson's disease D Sepsis E Herpes zoster

The Correct Answer is: D Choice D, sepsis, is one of a long list of disease entities that can cause troponin elevation, including arrhythmias (both tachycardic and bradycardic), aortic valve disease, hypertrophic cardiomyopathy, invasive cardiac surgeries and procedures, severe pulmonary hypertension, pulmonary embolism, myocardial infiltrative diseases (such as amyloidosis, sarcoidosis, scleroderma, and hemochromatosis), acute respiratory failure, burns, pericarditis, endocarditis, myocarditis, and even occasionally due to extreme athletic activities such as marathon running. Not included on this long list, however, are choices A, B, C, and E.

An 8-year-old girl is brought in to the emergency department with abdominal cramps, nausea, and vomiting since early this morning. She has had two loose stools but denies melena or hematochezia. She has had a low-grade fever. In the past hour, her vision has become blurry and she feels increasingly weak. Her mother has had similar but milder symptoms. Twenty-four hour dietary recall includes only chicken broth today. Last night for dinner they had meatloaf (fully cooked), mashed potatoes, and green beans. Her mother cans all their vegetables. Her medical history is unremarkable. She takes no medications. No known drug allergies. Examination reveals a temperature of 99°F, clear lungs, and mildly tachycardic heart with no murmur audible. Abdomen-bowel sounds present, soft with mild diffuse tenderness, no guarding. Neurologic examination is significant for decreased visual acuity and decreased motor strength (2/5) in the upper and lower extremities. The most likely etiology is A enterotoxic E coli B cholera species C pinworms D Clostridium botulinum

The Correct Answer is: D Clostridium botulinum produces a neurotoxin that can lead to life-threatening illness including respiratory paralysis. Botulism infection is caused by the spore-forming bacteria that lives in soil and can be foodborne. In the latter case, home-canned foods are often the cause. After a 12-hour to 3-day incubation period, botulism begins with classic symptoms of abdominal pain, nausea, vomiting, and mild diarrhea and, if unchecked, evolves into a progressive neurologic disorder marked by double vision, motor weakness, and ptosis. Respiratory muscle involvement may occur ultimately and result in death. Because of the virulence of the neurotoxin it has been used as an agent of bioterrorism. Cholera and enterotoxigenic E. coli cause a foodborne diarrheal illness that can result in significant morbidity and mortality, but they do not have neurologic manifestations. Pinworm infection is usually found among younger children, is marked by severe anal itching, and fecal-oral transmission.

A 46-year-old man with a history of EtOH abuse is brought to the emergency department in the morning by his wife. She has noted that he has developed tremors in both arms, and he seems mildly confused to her. He complains of feeling weak, with some cramping in the legs. On physical examination, his blood pressure is noted to be 162/95 mm Hg, and his heart rate is 108 beats/min. There is no asterixis. Which of the following electrolyte disorders are you likely to find in this patient? A hypercalcemia B hypocalcemia C hypermagnesemia D hypomagnesemia E hyperphosphatemia

The Correct Answer is: D Hypomagnesemia is a common finding in the patient who abuses alcohol. Other leading causes include diarrhea, diuretics, aminoglycosides, and amphotericin B. The etiology of hypomagnesemia in the patient with a history of alcohol abuse is thought to be a combination of malabsorption and inadequate dietary intake, possibly with alcohol exerting an antagonistic effect on absorption. Signs and symptoms are those of neuromuscular and central nervous system hyperirritability, including weakness and muscle cramps, tremors, nystagmus, a positive Babinski response, confusion, and disorientation. Hypertension, tachycardia, and ventricular arrhythmias may develop.

A 35-year-old man is brought to the emergency department with unremitting, generalized convulsive status epilepticus. The initial, preferred treatment is intravenous administration of which of the following? A phenobarbital B valproate C phenytoin D lorazepam E donepezil

The Correct Answer is: D In most patients suffering from generalized convulsive status epilepticus (GCSE), benzodiazepines such as lorazepam and diazepam are effective initial therapies due to their relatively high lipid solubility. As a result, they are able to cross the blood-brain barrier easily, which gives them the potential to stop seizures quickly. Lorazepam's lipid solubility is less compared to diazepam, and it also redistributes to fat more slowly. Hence, lorazepam tends to have a longer duration of action (12 to 24 hours) than diazepam (20 to 30 minutes). Phenytoin is often administered immediately after benzodiazepine administration for long-term seizure control, as it has a long half-life (20 to 36 hours) compared to diazepam. Phenytoin is not given first because its lipid solubility is less than the benzodiazepines and therefore cannot enter the brain quickly enough to terminate seizure activity.

A 62-year-old man with a history of hypertension, diabetes mellitus type 2, hyperlipidemia, and chronic tobacco use presents to the office with complaints of a retrosternal chest pressure associated with diaphoresis, nausea, and dyspnea, radiating down his left arm for the last 45 minutes after mowing his lawn. The patient's vital signs are stable, and on physical examination a new systolic murmur is appreciated. His EKG demonstrates evidence of acute anterolateral myocardial infarction on EKG, with ST segment elevation across the precordial leads, indicative of left anterior descending coronary artery stenosis. Which of the following is the most appropriate next step in management of this patient? A Dobutamine stress echocardiogram B Transthoracic echocardiogram C Nuclear stress test D Cardiac catheterization E Exercise treadmill stress test

The Correct Answer is: D In patients suffering from acute ST elevation myocardial infarction (STEMI), cardiac catheterization with percutaneous coronary intervention within 90 minutes substantially decreases morbidity and mortality outcomes; thus, choice D is the most appropriate next step in management of this patient. A transthoracic echocardiogram, choice B, will likely be performed during this patient's hospitalization, as it is can assist in assessment of complications of MI. However, the primary goal for acute STEMI is reperfusion in a timely manner, and thus TTE would not be the next step in the management of this patient. Choices A, C, and E are all forms of stress testing, which are useful testing modalities for patients with chest pain who are not actively infarcting.

A 54-year-old man presents to the emergency department with crampy abdominal pain, nausea, and vomiting. The patient has not passed gas or had a bowel movement for at least 10 hours. On examination, the abdomen is distended and there are high-pitched bowel sounds with rushes. A plain radiograph of the abdomen reveals cecal distension to 12 cm. What is the most appropriate definitive management for this patient? A Intravenous fluids B Nasogastric suction C Observation D Surgical exploration

The Correct Answer is: D Massive distention of the cecum, as detected on plain radiograph, is typically seen in "closed loop" obstructions where the ileocecal valve is competent. When distention approaches 12 cm, there is an increased risk of perforation and/or gangrene. Expedient surgical intervention is indicated. Although observation with intravenous fluids and nasogastric decompression are important adjuncts to management, surgical exploration is the only way to rapidly address this emergent situation.

A patient who is intoxicated presents to the emergency department. On ocular exam, you notice mydriasis. Which substance could he have been using? A sedatives B PCP C opioids D cocaine

The Correct Answer is: D Patients intoxicated with cocaine present with mydriasis. In opioid intoxication, the pupils are constricted. PCP intoxication is associated with nystagmus.

A 76-year-old male presents to the hospital with a hot, swollen, and painful right knee that appears to have an effusion. After your examination, you decide to perform an arthrocentesis. The fluid aspirate is turbid, cloudy, and most definitely looks like it is infectious material. Based on this clinical scenario, what would be most likely bacterial organism present in an elderly septic arthritis? A Hemophilus influenzae B Neisseria gonorrhoeae C Pseudomonas aeruginosa D Staphylococcus aureus E Streptococcus pneumonia

The Correct Answer is: D Septic arthritis is an infection within a joint space. The infection can be caused by multiple pathogens including bacteria, viruses, fungi and mycobacteria, but in older adults the most common organism implicated is Staphylococcus aureus. These bacteria can get into the joint space through multiple mechanisms including direct inoculation after a penetrating injury, hematogenous spread from an infection elsewhere in the body, or from extension from a contiguous bone infection. All of the bacteria mentioned as possible answers are possible causes of septic arthritis, but there prevalence varies within different patient populations. Hemophilus influenza and Streptococcus pneumonia are commonly seen in children less than five years of age that develop septic arthritis. Neisseria gonorrhoeae is most common in sexually active adolescents and young adults. Pseudomonas aeruginosa is a common cause of septic arthritis in IV drug abusers.

A 14-year-old boy presents to the emergency department with acute scrotal pain and vomiting for the past 2 hours. His left testicle is in extreme pain and he states the pain started while playing basketball in gym class. On physical exam of the affected testicle, which of the following findings would suggest testicular torsion? A Transillumination B Positive Prehn's sign C Positive cremasteric reflex D Abnormal transverse lie E Mass of enlarged veins palpated

The Correct Answer is: D Testicular torsion is most common between ages 12-18 with the classic presentation of abrupt and severe onset of pain with nausea/vomiting. The testicle on physical examination is painful, swollen, high-riding, tender, and has an abnormal transverse lie (D). Transillumination (A) is when light is placed behind the scrotum and fluid is illuminated in cases of hydroceles. Prehn's sign (B) is pain relief with elevating the scrotum and is positive in cases of epididymitis. The cremasteric reflex (C) is a normal finding that causes elevation of the testis on the ipsilateral side when the inner aspect of the inner thigh is stroked. The absence of cremasteric reflex on the affected side is often found in acute torsion. A mass of enlarged veins (E), or "bag of worms," is a finding associated with a varicocele.

An 18 month old female is brought to the pediatricians office with a history of cough, fever of 102, and decreased fluid intake. Her immunizations are not up to date as the family just moved to the United States from out of the country. On physical exam she is drooling and sitting up in a "tripod position" with mild stridor. What is the most appropriate treatment indicated for this condition? A Humidified air B Albuterol nebulizer C Budesonide nebulizer D Recemic epinephrine nebulizer E Ipratropium nebulizer

The Correct Answer is: D The clinical presentation suggests epiglotitis. This is an emergent airway condition. The anesthesiologist , or the pediatric otolaryngologist must be called to stand by to intubate or insert a tracheostomy if the patients airway closes. Racemic epinephrine via nebulizer relieves much of the edema to the upper airway in a patient with epiglotitis. It is a stabilizing measure until definitive care can be arranged. Oxygen and antibiotics should administered emergently also. No x-rays are indicated when the presentation is classic. Albuterol is a beta-agonist used for treatment of asthma. Budesonide,a steroid and ipratropium, an anticholinergic agent are most often used in combination with albuterol for treatment emphysema and asthma.

A 23-year-old man, unrestrained driver, is brought to the emergency department by ambulance after having been involved in an automobile accident. His vitals are BP: 99/54 mm Hg, P: 112/min, R: 18/min, oxygen saturation: 99%, T: 99.8°F. Examination reveals mild abdominal tenderness with pain radiating to the right shoulder. What is the most appropriate diagnostic test to order initially? A Computed tomography of the abdomen and pelvis B Diagnostic peritoneal lavage C Flat and upright abdominal radiographs D Diagnostic ultrasound

The Correct Answer is: D The initial evaluation of blunt abdominal trauma is by the performance of a FAST (focused assessment with sonography for trauma) ultrasound, which is performed by an emergency department physician or surgeon. CT scan remains an adjunct test in hemodynamically stable patients or in patients in whom further assessment of solid intra-abdominal organs is required.

A 22-year-old woman, with no previous medical problems, suddenly cried out, fell to the ground, extended her legs, flexed her arms, and jerked her extremities for 30 seconds. There was associated tongue biting and urinary incontinence. She awoke slowly over a 10-minute period and recalled nothing about the episode. She remained lethargic for several hours but the rest of her neurologic examination was normal. What is the most likely etiology for this episode? A epilepsy B hyperventilation C cardiac arrhythmia D seizure E stroke

The Correct Answer is: D This event represents a well-demarcated episode affecting some combination of consciousness, motor, and/or sensory function consequent to abnormal electrical discharges in the brain. This is consistent with the definition of a seizure. Epilepsy refers to multiple, recurrent seizures. This history is not consistent with hyperventilation, stroke, or cardiac arrhythmia, which would typically include chest pain, shortness of breath, dyspnea on exertion, or focal neurological deficits.

A 44-year-old female presents to the emergency department with a right-sided headache. She states the headache is located on the right temple region, is non-radiating, and does not cause photophobia. She is otherwise healthy and has no reported medical problems and has only taken acetaminophen for the pain, which has minimal relief. On physical exam she is alert, awake, and oriented. Her vitals are T 98.8, P 78, R 18, and BP 128/76. Her head is normocephalic, atraumatic, and pupils are equal and reactive. She has tenderness to the right temporal area of the temporal artery, and there is no noted swelling, redness, or abnormalities noted. There is no tenderness to the cervical muscles. She exhibits a non-focal neurological exam. Based on the clinical presentation, what test is the best test in confirming an accurate diagnosis? A C-reactive protein B Erythrocyte sedimentation rate (ESR) C Computed tomography of head D Temporal artery biopsy E Complete blood count (CBC)

The Correct Answer is: D This patient is exhibiting a case of temporal arteritis. She has the signs and symptoms that are classic in nature and do not fit into the realm of the other headaches, all of which usually have different characteristics on history and physical exam. The best test to confirm the diagnosis is to biopsy the temporal artery (D) for pathologic confirmation. The three blood tests (A, B, and E) would only suggest the presence of the disease, but they are not specific enough to confirm. Computed tomography of the head (C) would not be indicated as temporal arteritis is an extracranial condition.

A 66-year-old male with a history of hypertension, diabetes mellitus, and hypercholesterolemia presents by emergency medical services (EMS) to the emergency department complaining of severe chest pain with radiation into his back. The patient states that he was feeling well in the morning, but while performing some light activity he felt a "ripping" sensation in his back, which he initially thought was a pulled muscle. The pain continued and the patient started to have chest pain, shortness of breath, and lightheadedness. On initial examination the patient is still in pain, pale, diaphoretic, and has a blood pressure of 85/40. His chest is clear to auscultation, and he has a 3/6 diastolic murmur best appreciated at the base of the heart. Given this clinical scenario, what would be the expected finding on chest x-ray? A Normal B Pleural effusion C Decreased lung volume D Widened mediastinum E Cardiomegaly

The Correct Answer is: D This patient is exhibiting a history and physical examination that is consistent with a thoracic aneurysm. The patient's history of hypertension, along with the "ripping" sensation in his back and hypotension give a clinical presentation that is suggestive of a thoracic aneurysm dissection. In this instance it would be expected that the patient would have evidence of a widened mediastinum (D) on chest radiography. While pleural effusions (B), decreased lung volumes (C), and cardiomegaly (E) can all be present on a chest film with an aneurysm, the widened mediastinum would be the expected finding if you suspect an aneurysm. It is very unlikely that a thoracic aneurysm with hypotension would have a normal chest radiograph finding (A).

A 55-year-old right-hand dominant man presents with a 4-hour history of weakness and tingling of his right hand and numbness of the right side of his mouth. Mild difficulty was noted with word finding. His symptoms have improved since onset but have not fully resolved. There is no significant medical history. Physical examination revealed flat right nasolabial fold, subjective numbness of the right hand, right pronator drift, clumsiness of finger tapping on the right hand, increased deep tendon reflexes on the right, as well as a present Babinski. What is the most likely etiology for this patient's problem? A migraine headache B peripheral neuropathy C syncope D transient ischemic attack E seizure

The Correct Answer is: D Three key features of a transient ischemic attack include sudden onset and complete reversal of symptoms within 24 hours, usually within 15 minutes. The symptoms are usually in the anatomical distribution of a single blood vessel. This patient's history is not suggestive of migraine or syncope. His physical examination findings do not correlate with peripheral neuropathy or seizure.

A 65-year-old recent alcoholic comes to the emergency department with recent onset of dyspnea, with exertion, 3 pillow orthopnea, lower extremity edema, and palpitations, in which he describes his heart as racing. Which of the following is most likely to be the cause of his high-output congestive heart failure? A Mitral regurgitation B Aortic stenosis C Uncontrolled hypertension D Ruptured chordae tendinae E Beriberi

The Correct Answer is: E Choice E, beriberi, also known as thiamine deficiency, is common among alcoholics, and the only high-output cause of congestive heart failure among the choices offered. Other causes include severe anemia, thyrotoxicosis, and arteriovenouis shunting (for example, in hemodialysis patients). Choice A, mitral regurgitation, is a cause of excessive preload, leading to heart failure. Choice D, ruptured chordate tendinae associated with mitral regurgitation, would also be a cause of excessive preload, leading to heart failure. Choices B and C, aortic stenosis and uncontrolled hypertension, are causes in which too much afterload leads to heart failure.

A 22-year-old recent immigrant from Vietnam, who is 28 weeks pregnant with her first child, presents to the emergency department with complaints of worsening dyspnea and lower extremity edema. She is unable to answer definitively whether or not she has a history of rheumatic fever. On physical examination, a possible opening snap, loud S 1 , and a very soft diastolic rumbling murmur is auscultated. When the patient is placed in the left lateral decubitus position, the murmur is accentuated, and heard best at the apex. With inspiration, the murmur does not increase in amplitude. Which of the following is the most likely finding on echocardiogram? A Tricuspid regurgitation B Tricuspid stenosis C Atrial septal defect D Aortic regurgitation/insufficiency E Mitral stenosis

The Correct Answer is: E Choice E, mitral stenosis, is the most likely finding in this patient, who presents with physical exam findings including a possible opening snap, loud S 1 , and a very soft diastolic rumbling murmur which is heard best at the cardiac apex and accentuated by placing the patient in the left lateral decubitus position. Although rheumatic fever was not positively confirmed, the patient likely did have a history, given that the majority of cases of mitral stenosis are secondary to rheumatic heart disease. Patients from Asia, Central America, and South America are exposed more frequently than their counterparts in more developed countries, where antibiotic use is more common. Choices A and B, tricuspid regurgitation and tricuspid stenosis, are also linked with patients with rheumatic heart disease. The murmur of tricuspid regurgitation, however, is a systolic murmur, which increases with inspiration and is heard best at the left lower sternal border. Tricuspid stenosis presents with a diastolic murmur, and with inspiration the murmur increases. It, too, is heard best at the left lower sternal border. Choice C, an atrial septal defect, if large, could present with similar symptoms of exertional dyspnea secondary to a large shunt, but auscultation would reveal a moderately loud systolic ejection murmur that is heard best in the second and third interspaces. This is secondary to increased pulmonary arterial flow. Choice D, aortic regurgitation/insufficiency, is also a diastolic murmur; however, it is usually a diastolic decrescendo murmur that is heard best at the left sternal border.

A 46-year-old female with a history of poorly-controlled diabetes and grand mal seizures presents for evaluation of bilateral foot pain. She describes the pain as burning and has noticed it is worse at night. She occasionally has this pain in her feet. She denies other medical conditions, and her medications are metformin and dilantin. Which of the following medications, used for the suspected diagnosis, should be avoided in this patient? A Capsaicin B Desipramine C Gabapentin D Lidocaine patch E Tramadol

The Correct Answer is: E Diabetic peripheral neuropathy is the diagnosis, and describes any neuropathy in the diabetic patient. This patient is exhibiting a distal symmetric polyneuropathy with the classic associated symptoms, commonly called "pins and needles" by patients. When associated with pain and functional impact, pharmacologic therapy is warranted. There are many agents to choose from, with each of the answer choices being options. However, in a patient with a known seizure disorder, tramadol should be avoided, as it decreases the seizure threshold. Gabapentin, also a seizure medication, may be used, but close monitoring is suggested.

A 59-year-old woman presents to the emergency department with an acute upper gastrointestinal hemorrhage. Her medical history is pertinent for peptic ulcer disease for the past 5 years and hypertension. A nasogastric tube is inserted and bright red blood is seen. Her vital signs are BP: 110/70 mm Hg, P: 94/min, R: 14/min, oxygen saturation: 97%, T: 99°F. Which of the following diagnostic studies would be the most appropriate next step to determine the site of bleeding? A Abdominal and pelvic computed tomography B Abdominal ultrasound C Upper gastrointestinal series with barium D Bleeding scan E Upper endoscopy

The Correct Answer is: E Patients who present with upper gastrointestinal bleeding and shock requiring multiple transfusions in 24 hours are at high risk for mortality from gastrointestinal bleeding. The upper gastrointestinal bleeding in this patient warrants further investigation with upper gastrointestinal endoscopy to both determine the site of bleeding and provide potential therapy by endoscopic electrocautery or injection.

A 23-year-old female has been seen in your Emergency Department after being beaten by her husband. As you counsel her before she leaves, which of the following will you recommend? A Avoiding the behavior that brought on the attack B Couples counselling C Leaving the relationship D Prosecuting her husband E Referral to a local women's shelter

The Correct Answer is: E The appropriate course of action when working with a person who has suffered intimate partner violence is to validate his or her experience, document clearly and non-judgmentally, and assess immediate safety. Referrals to appropriate resources (E) should be made, but decisions regarding the relationship (B, C) and any legal action (D) should be left to the patient rather than continuing a pattern of controlling behavior. Suggesting she avoid behaviors that provoke her attacker (A) puts the blame on the victim.

A 39-year-old man presents to the emergency department with massive hematemesis. His physical examination reveals slight jaundice, palmar erythema, spider angiomas, and marked ascites. Vitals at the time of presentation are as follows: BP: 85/44 mm Hg, P: 122/min, R: 16/min, oxygen saturation: 96%, and T: 99.8°F. Which of the following is the most likely cause of the massive hematemesis? A Peptic ulcer disease B Mallory-Weiss tear C Gastric carcinoma D Arteriovenous malformation E Esophageal varices

The Correct Answer is: E The most common cause of massive upper gastrointestinal bleeding in patients with cirrhosis is esophageal varices. Although 20% of patients with portal hypertension will have bleeding from other causes (peptic ulcer disease, Mallory-Weiss tears, or gastritis), endoscopic evaluation in patients with portal hypertension is necessary for diagnosis and initial therapy.

A 32-year-old man who is HIV positive has a seizure. On presentation to the emergency department (ED) he is confused and unsure of what happened. His partner reports that he had been complaining of headache in the days preceding the event. CT scanning of the head demonstrates five peripheral contrast-enhancing lesions. What is the most likely diagnosis? A AIDS dementia complex B central nervous system lymphoma C cryptococcal meningitis D progressive multifocal leukoencephalopathy E toxoplasmosis

The Correct Answer is: E The most common space-occupying CNS lesion in patients with HIV is toxoplasmosis. This condition may present with headache, focal neurologic deficits, seizures, and/or mental status changes. The typical appearance on brain imaging is that of multiple contrast-enhancing lesions in the periphery, particularly the basal ganglia. CNS lymphoma is more typically a single lesion. AIDS dementia complex presents a diagnosis of exclusion, without a characteristic appearance on imaging. The diagnosis of cryptococcal meningitis is made by examination of the spinal fluid, while PML imaging shows nonenhancing white matter lesions without mass effect.

A 52-year-old male with chronic alcoholism is brought to the emergency department by his family, due to his acting differently for several days. A physical exam reveals nystagmus, eye muscle weakness, global confusion, retrograde amnesia, and ataxia. Which of the following is the most likely etiology of the diagnosis? A Cerebrovascular accident B Hypoxemia C Uremia D Vitamin B12 deficiency E Vitamin B1 (thiamine) deficiency

The Correct Answer is: E This patient exhibits classic symptoms associated with Wernicke encephalopathy. Wernicke encephalopathy is due to vitamin B1 (thiamine) deficiency. In the United States, this condition occurs most frequently in chronic alcoholics, but it may be seen in any condition affecting thiamine levels. Each of the other etiologies listed may also cause neurologic findings, and should be considered in the differential diagnosis and evaluation of this patient.

A 1-year-old boy is brought to the emergency department by his parents, who state that the child refuses to walk or crawl and begins crying when they stand him. He seems calm while lying on the examination table. Vitals are as follows: Temp: 38°C, HR: 70, RR: 15. Bruising is noted in several places. His parents deny trauma, but have noticed that he bruises easily. What other physical finding would you expect? A Conjunctival hemorrhages secondary to shaken baby syndrome B Pain response over the wrists secondary to passive range of motion C Pain response with passive range of motion to the hip secondary to slipped epiphysis D Pharyngitis and sand paper rash secondary to a staph infection E Swelling and warmth over the knee secondary to hemarthroses

The Correct Answer is: E This patient has hemophilia A. Hemarthroses usually occur when an affected child begins to walk. Due to his hemophilia, easy bruising can occur. Hemarthroses can cause low-grade fevers without infection being present, so choice D is incorrect. Wrist joints are less involved then knees, ankles, and elbows.

A 42-year-old woman with a history of migraine cephalgia and Raynaud's phenomenon comes to the emergency department with complaints of severe chest discomfort that occurs at rest every morning (at approximately 10 AM). An EKG performed during an episode of chest discomfort demonstrates transient ST segment elevation, which is relieved with sublingual nitroglycerin. There is no troponin elevation. Cardiac catheterization is performed, and reveals coronary artery spasm, which corresponds with ST segment elevation, and no significant coronary artery stenosis. Which of the following is the most appropriate treatment regimen? A 24-hour nitroglycerin dermal patch B Thiazide diuretics C Loop diuretics D Calcium channel blockers E Aspirin

he Correct Answer is: D Choice D, calcium channel blockers, and long-acting nitrate therapy have been proven to be effective for preventing recurrences of episodes of Prinzmetal angina, with short-acting sublingual or IV nitroglycerin useful for relieving acute episodes. Choice A, 24-hour nitroglycerin dermal patch, is inappropriate, as patients can develop nitrate tolerance, and thus need a 12-hour nitrate-free period every day. Choices B and C, thiazide and loop diuretics, have no proven benefit in patients with Prinzmetal angina. Aspirin, choice E, may worsen episodes of prinzmetal angina, and thus is not recommended.


Related study sets

Ch. 29 Assessment of Hematologic System

View Set

Unit 6: Spatial data collection methods

View Set

CH 28 DISORDERS OF CARDIAC CONDUCTION& RHYTHM

View Set

English Phrasal verbs in Use (intermediate)

View Set

MKTG303 Exam 1 Study Guide (chapters 1-8)

View Set